4
$\begingroup$

Let $a$ and $b$ be two points in the plane. Let's choose a point $c$ uniformly from the circle of radius $r$ with $a$ as center and choose a point $d$ uniformly from the circle of radius $r$ with $b$ as the centre of this circle. Let $X$ be the eucleadian distance between $c$ and $d$. What's the density function of $X$? Is it hard compute? P.S. I know how to do it when $a=b$.

$\endgroup$
5
  • $\begingroup$ MO is for math research questions. Is there a research angle to this question? $\endgroup$ Nov 11, 2015 at 12:05
  • $\begingroup$ Yes Gerry, I want to compute average degree of nodes placed, roughly, in a grid where adjacency defined in terms of distance. Problem arises in sensor networks. $\endgroup$
    – Mudi
    Nov 11, 2015 at 14:19
  • 1
    $\begingroup$ Do you mean the points are drawn from the interiors or from the boundaries? $\endgroup$ Nov 11, 2015 at 15:48
  • 1
    $\begingroup$ What do you get when $a=b?$ $\endgroup$
    – Igor Rivin
    Nov 11, 2015 at 16:39
  • $\begingroup$ Yes, I am sorry I meant to choose uniformly from the interior of circle. When $a=b$, the expression doesn’t look pretty (not to me at least) but the argument/procedure is simple enough (It’s described in Tuckwell’s Applications of Probability Theory). Anyway here it is. $f_X(x) = \frac{2x}{\pi r^2}( 2 \arccos(\frac{x}{2r}) - \frac{x}{r} \sqrt{ 1-{(\frac{x}{2r}) }^2} )$. $\endgroup$
    – Mudi
    Nov 11, 2015 at 17:17

1 Answer 1

2
$\begingroup$

Empirically, the distribution appears approximately normal. Below is the distribution for $10^6$ pairs of $c,d$-points, $r=1$, $\| a - b \|=3$ (so the closet $X=\| c - d \|$ distance possible is $1$, and the furthest distance possible is $5$.


          DistanceDistribution


$\endgroup$
2
  • $\begingroup$ Seeing @DouglasZare's question, I assumed that "from the circle" and "in unit circles" mean from the interior of the circles (rather than on the circle boundaries). $\endgroup$ Nov 11, 2015 at 16:29
  • $\begingroup$ This is interesting - it didn't occur to me to check it empirically. Can you think of a way to prove this hypothesis? Thanks. $\endgroup$
    – Mudi
    Nov 11, 2015 at 17:32

Your Answer

By clicking “Post Your Answer”, you agree to our terms of service and acknowledge you have read our privacy policy.

Not the answer you're looking for? Browse other questions tagged or ask your own question.